LSAT and Law School Admissions Forum

Get expert LSAT preparation and law school admissions advice from PowerScore Test Preparation.

User avatar
 Dana D
PowerScore Staff
  • PowerScore Staff
  • Posts: 132
  • Joined: Feb 06, 2024
|
#105756
Complete question explanation

Assumption question. The correct answer is C.


The doctor concludes that the cortisone injection resulted in significant pain relief.

Because the doctor is introducing a causal relationship, we also know the doctor is saying without question that there are no alternative causes for the pain relief other than the corisone.

Answer Choice (A) This does not have to be true. Computerized scans might not be the best way, but they still could be sufficient for this diagnosis.

Answer Choice (B)The doctor does not assume cortisone reduced the pressure, only that the cortisone reduced the inflamed nerve, which was causing the pain. The pressure might still be there and the nerve could get inflamed again, which only strengthens the doctors argument.

Answer Choice (C) This is the correct answer.
If patient's belief in the efficacy of the cortisone resulted in pain relief, than a placebo could be given to the patient and the pain relief still would have occurred. In other words, the Effect would have occurred without the Cause.

Answer Choice (D) Other treatments may reduce inflammation - the doctor only says the 'best' way is a cortisone injection.

Answer Choice (E) This is essentially what the doctor already explicitly says - the best treatment choice was a cortisone shot.
 blake123
  • Posts: 1
  • Joined: Nov 17, 2019
|
#72079
Hello,

I have a more broad question about Necessary assumption questions but also relates directly to this question. The correct answer C ("The pain relief did not occur merely through the patient's belief in the efficacy of the cortisone") mentioned information that was not directly said in the stimulus. I thought necessary assumption questions had to have a correct answer that stated information directly from the stimulus and did not include any new information? If that is the case, how can C be the correct answer then because at no point does the argument mention anything about a "patient's belief?"

Thank you!
User avatar
 Dave Killoran
PowerScore Staff
  • PowerScore Staff
  • Posts: 5853
  • Joined: Mar 25, 2011
|
#72082
blake123 wrote:I thought necessary assumption questions had to have a correct answer that stated information directly from the stimulus and did not include any new information?
Hi Blake,

The above quoted section is not accurate. You don't need a repeat of info from the stimulus, you need to find the "missing link" or or knock out something that would attack the argument. There are a ton of resources I can point you to that discuss this idea, from our books to our courses to our podcast, so let me know what you material you have of ours and I'll try to point you in the right direction.

Thanks!
 medialaw111516
  • Posts: 80
  • Joined: Dec 11, 2018
|
#72156
I'm confused about why B is wrong ( I think I don't understand what the argument is actually saying) and C just seems so random even though I know we're looking for a missing link.
 Serah
  • Posts: 2
  • Joined: Nov 04, 2019
|
#72301
medialaw111516 wrote:I'm confused about why B is wrong ( I think I don't understand what the argument is actually saying) and C just seems so random even though I know we're looking for a missing link.
I have the same question!
User avatar
 KelseyWoods
PowerScore Staff
  • PowerScore Staff
  • Posts: 1079
  • Joined: Jun 26, 2013
|
#72317
Hi Medialaw111516 & Serah!

As Dave mentioned, in our books and courses we cover 2 types of assumptions: 1) assumptions that provide a missing link between the premises and the conclusion, and 2) assumptions that knock out something that would attack the argument. In this case, the assumption they give us is not a missing link assumption; instead, it defends the argument from a possible source of attack.

This is a causal argument. The doctor says that he gave the patient a cortisone injection in this specific nerve, the pain was relieved, so the doctor concludes that this specific nerve was causing the patient's pain. Basically the doctor thinks that reducing the nerve inflammation is what caused the pain to be relieved. But what if there's some other explanation for the patient's pain relief? What if there was something else the cortisone shot could have affected besides the inflammation of that specific nerve?

Answer choice (C) defends the argument against a possible source of attack by eliminating a potential alternate cause of the patient's pain relief. It tells us that the patient's pain relief was not simply caused by the placebo effect. Our Assumption Negation Technique really helps with this one: If you take the opposite of answer choice (C) so that it instead reads that it was the patient's belief in the efficacy of the cortisone that caused the pain relief, that would kill this argument that the inflammation of the nerve was the cause of the pain.

Try out that Assumption Negation Technique on answer choice (B) and you can see that it isn't necessary for the argument. If I take the opposite of (B), it would read that the cortisone injection reduced pressure on the inflamed nerve. That doesn't attack the argument that the nerve is the cause of the pain, that supports it!

If Assumption questions are giving you trouble, I definitely recommend you look into our Assumption Negation Technique. It's super helpful with this tricky question type!

Best,
Kelsey
User avatar
 giulicard
  • Posts: 2
  • Joined: Jan 12, 2021
|
#83229
Kelsey, thanks so much for taking the time to explain it, though I honestly don't think I could have gotten this right regardless of your explanation. For this specific section, I really fell off after question 19... I got most wrong after this question! Anyone else?
User avatar
 Taisiya
  • Posts: 10
  • Joined: Mar 12, 2021
|
#89487
giulicard wrote: Wed Jan 13, 2021 6:36 pm Kelsey, thanks so much for taking the time to explain it, though I honestly don't think I could have gotten this right regardless of your explanation. For this specific section, I really fell off after question 19... I got most wrong after this question! Anyone else?
Totally late to the thread but, right there with you! Most were wrong. Maybe the timing wore us out? Going to hope that practice (and review) makes perfect:D

Get the most out of your LSAT Prep Plus subscription.

Analyze and track your performance with our Testing and Analytics Package.